A coffee distributor needs to mix a(n) Costa Rican coffee blend that normally sells for $9.10 per pound with a Arabian Mocha coffee blend that normally sells for $13.10 per pound to create 100 pounds of a coffee that can sell for $11.58 per pound. How many pounds of each kind of coffee should they mix?

Answers

Answer 1

9514 1404 393

Answer:

38 pounds Costa Rican62 pounds Arabian Mocha

Step-by-step explanation:

Let 'a' represent the number of pounds of Arabian Mocha in the mix. Then the number of pounds of Costa Rican blend is (100-a). The cost of the mix will be ...

  9.10(100 -a) +13.10(a) = 11.58(100)

  4a = 248 . . . . . . . . . . . . . collect terms, subtract 910

  a = 62 . . . . . . . . . . . . divide by 4

62 pounds of Arabian Mocha and 38 pounds of Costa Rican blend should be mixed.


Related Questions

A projectile is fired from ground level with an initial velocity of 35 m/s at an angle of 35° with the horizontal. How long
will it take for the projectile to reach the ground?

Answers

Answer:

Step-by-step explanation:

We will work in the y-dimension only here. What we need to remember is that acceleration in this dimension is -9.8 m/s/s and that when the projectile reaches its max height, it is here that the final velocity = 0. Another thing we have to remember is that an object reaches its max height exactly halfway through its travels. Putting all of that together, we will solve for t using the following equation.

[tex]v=v_0+at[/tex]

BUT we do not have the upwards velocity of the projectile, we only have the "blanket" velocity. Initial velocity is different in both the x and y dimension. We have formulas to find the initial velocity having been given the "blanket" (or generic) velocity and the angle of inclination. Since we are only working in the y dimension, the formula is

[tex]v_{0y}=V_0sin\theta[/tex] so solving for this initial velocity specific to the y dimension:

[tex]v_{0y}=35sin(35)[/tex] so

[tex]v_{0y}=[/tex] 2.0 × 10¹ m/s

NOW we can fill in our equation from above:

0 = 2.0 × 10¹ + (-9.8)t and

-2.0 × 10¹ = -9.8t so

t = 2.0 seconds

This is how long it takes for the projectile to reach its max height. It will then fall back down to the ground for a total time of 4.0 seconds.

PLEASE HELP, according to this function, which is a true statement???????

Answers

Answer:

function ar true in the mach on which

Answer:

i think three one is right answer...

Given a parametric curve

{x = 2 cost
{y = 4 sint 0 <= t <= π

a. Set up but do NOT evaluate an integral to find the area of the region enclosed by the curve and the x-axis.
b. Set up but do NOT evaluate an integral to find the area of the surface obtained by rotating the curve about the x-axis.

Answers

(a) The area of the region would be given by the integral

[tex]\displaystyle\int_0^\pi y(t)\left|x'(t)\right|\,\mathrm dt = 8 \int_0^\pi \sin^2(t)\,\mathrm dt[/tex]

(b) The area of the surface of revolution would be given by

[tex]\displaystyle\int_0^\pi y(t)\sqrt{x'(t)^2+y'(t)^2}\,\mathrm dt = 4\int_0^\pi\sin(t)\sqrt{4\sin^2(t)+16\cos^2(t)}\,\mathrm dt[/tex]

Please help on my hw, I'm not feeling good, and can't concentrate

Answers

Answer:

Solution given:

f(x)=x²

g(x)=x+5

h(x)=4x-6

now

23:

(fog)(x)=f(g(x))=f(x+5)=(x+5)²=x²+10x+25

24:

(gof)(x)=g(f(x))=g(x²)=x²+5

25:

(foh)(x)=f(h(x))=f(4x-6)=(4x-6)²=16x²-48x+36

26:

(hof)(x)=h(f(x))=h(x²)=4x²-6

27;

(goh)(x)=g(h(x))=g(4x-6)=4x-6+5=4x-1

28:

(hog)(x)=h(g(x))=h(x+5)=4(x+5)-6=4x+20-6=4x-14

Which expression gives the best estimate of 30 percent of 61?

The answers are below:

Hurry, please!

Answers

Answer:

it would be 1/4(60)

Step-by-step explanation:

30 percent of 61 is 18.3 and 1/4 of 60 is 15 which is closest to 18.3

Roulette is a casino game that involves spinning a ball on a wheel that is marked with numbered squares that are red, black, or green. Half of the numbers 1 - 36 are colored red and half are black and the numbers 0 and 00 are green. Each number occurs only once on the wheel. What is the probability of landing on an even number and a number greater than 17? (A number is even if it is divisible by 2. 0 and 00 are considered even as well.)​

Answers

Answer:

the wording (punctuation) of the question can lead to different interpretations....

I assume that the question was >17 & even which is "5/19",

BUT... it can also be read as two questions

first >17 which is "10/19"

and second an even number which is "9/19"

BUT !!! I think that the question answer is 5/19

Step-by-step explanation:

Even Number  = 18/38 = 9/19

greater 17 = 20/38 = 10/19

Even & greater 17 = 10/38 = 5/19

If F is the function defined by F(x)=3x−1, find the solution set for F(x)=0.

Answers

The solution for set F(x) is -1

Approximate 5.7255 to the nearest thousand

Answers

Step-by-step explanation:

round 5.7255 to thousands place

place after thousands place (5) rounds up the 5 before it

therefore 5.726 ur ans

MARK above ANS as branliest

Will give brainliest answer, there has to be two answers to give one of you the brainliest

Answers

Answer:

C

Step-by-step explanation:

[tex] ({8}^{ { - 9})^{ \frac{ - x}{9} } } [/tex]

when putting a power to another power, then these two powers are multiplied.

so, -9 × (-x/9) = (-9 × -x) / 9 = 9x/9 = x

so, this reduces to the original

[tex] {8}^{x} [/tex]

and therefore C is the right answer.

F(x) =-2x-4 find x if f(x)=14

Answers

Answer:

14=-2x-4

18=-2x

x=-9

Hope This Helps!!!

A courier service company wishes to estimate the proportion of people in various states that will use its services. Suppose the true proportion is 0.06. If 235 are sampled, what is the probability that the sample proportion will differ from the population proportion by greater than 0.04

Answers

Answer:

0.0098 = 0.98% probability that the sample proportion will differ from the population proportion by greater than 0.04

Step-by-step explanation:

To solve this question, we need to understand the normal probability distribution and the central limit theorem.

Normal Probability Distribution:

Problems of normal distributions can be solved using the z-score formula.

In a set with mean [tex]\mu[/tex] and standard deviation [tex]\sigma[/tex], the z-score of a measure X is given by:

[tex]Z = \frac{X - \mu}{\sigma}[/tex]

The Z-score measures how many standard deviations the measure is from the mean. After finding the Z-score, we look at the z-score table and find the p-value associated with this z-score. This p-value is the probability that the value of the measure is smaller than X, that is, the percentile of X. Subtracting 1 by the p-value, we get the probability that the value of the measure is greater than X.

Central Limit Theorem

The Central Limit Theorem establishes that, for a normally distributed random variable X, with mean [tex]\mu[/tex] and standard deviation [tex]\sigma[/tex], the sampling distribution of the sample means with size n can be approximated to a normal distribution with mean [tex]\mu[/tex] and standard deviation [tex]s = \frac{\sigma}{\sqrt{n}}[/tex].

For a skewed variable, the Central Limit Theorem can also be applied, as long as n is at least 30.

For a proportion p in a sample of size n, the sampling distribution of the sample proportion will be approximately normal with mean [tex]\mu = p[/tex] and standard deviation [tex]s = \sqrt{\frac{p(1-p)}{n}}[/tex]

Suppose the true proportion is 0.06.

This means that [tex]p = 0.06[/tex]

235 are sampled

This means that [tex]n = 235[/tex]

Mean and standard deviation:

[tex]\mu = p = 0.06[/tex]

[tex]s = \sqrt{\frac{p(1-p)}{n}} = \sqrt{\frac{0.06*0.94}{235}} = 0.0155[/tex]

What is the probability that the sample proportion will differ from the population proportion by greater than 0.04?

Proportion below 0.06 - 0.04 = 0.02 or above 0.06 + 0.04 = 0.1. Since the normal distribution is symmetric, these probabilities are equal, which means that we can find one of them and multiply by 2.

Probability the proportion is below 0.02.

p-value of Z when X = 0.02. So

[tex]Z = \frac{X - \mu}{\sigma}[/tex]

By the Central Limit Theorem

[tex]Z = \frac{X - \mu}{s}[/tex]

[tex]Z = \frac{0.02 - 0.06}{0.0155}[/tex]

[tex]Z = -2.58[/tex]

[tex]Z = -2.58[/tex] has a p-value of 0.0049.

2*0.0049 = 0.0098

0.0098 = 0.98% probability that the sample proportion will differ from the population proportion by greater than 0.04

There are seven people online and each person uses either a blue, black, or red pen to write while working. How many possible combinations can be made?


A. 49

B. 28

C. 21

D. 10


A S A P!! I DONT WANT MY DAD YELLING AT ME FOR THIS ;^;

Answers

Answer:

21 is the answer for this question may be

Algebraically show that each of the given combinations are equivalent to the given functions. f(x) – g() is
equivalent to m(x) given:
f(0)
= - 3x + 5; g(x)
- 5x – 7; m(x) = 2x + 12
f(x) – g(x) = (
=
Is f(x) – g(x) equivalent to m(x)? yes

Answers

Answer:

[tex]f(x) - g(x) = 2x + 12[/tex]

[tex]m(x) = f(x) - g(x)[/tex] --- True

Step-by-step explanation:

Given

[tex]f(x) = -3x + 5[/tex]

[tex]g(x) = -5x - 7[/tex]

[tex]m(x) = 2x + 12[/tex]

Solving (a): [tex]f(x) - g(x)[/tex]

From the given parameters, we have:

[tex]f(x) = -3x + 5[/tex]

[tex]g(x) = -5x - 7[/tex]

So:

[tex]f(x) - g(x)=-3x+5 + 5x + 7[/tex]

Collect like terms

[tex]f(x) - g(x) = 2x + 12[/tex]

Solving (b) m(x) = f(x) = g(x)?

In (a), we have:

[tex]f(x) - g(x) = 2x + 12[/tex]

And

[tex]m(x) = 2x + 12[/tex] --- given

By comparison:

[tex]m(x) = f(x) - g(x)[/tex]

CAN SOMEONE PLEASE HELP

A six sided number cube rolled once. what is the probability of landing on a multiple of 2. write the probability as a fraction, percent and decimal.

probability (as fraction)=
probability (as percent)=
probability (as decimal)=

Answers

Answer:

P( fraction) = 1/2

P ( percent) = 50%

P ( decimal) = .5

Step-by-step explanation:

The possible outcomes on  a six sided cube are 1,2,3,4,5,6

Multiples of 2 are  2,4,6

P( multiple of 2) = number of multiples of 2 / total outcomes

                           = 3/6 = 1/2

P( fraction) = 1/2

P ( percent) = 50%

P ( decimal) = .5

A large container holds 4 gallons of chocolate milk that has to be poured into bottles. Each bottle holds 2 pints.
If the ratio of gallons to pints is 1: 8,
bottles are required to hold the 4 gallons of milk.

Answers

Answer:

64 Bottles

Step-by-step explanation:

that is the procedure above

For what values of the variable, do the following fractions exist: y^2-1/y+y/y-3

For what values of the variable, do the following fractions exist: b+4/b^2+7

For what values of the variable, do the following fractions exist: a/a(a-1)-1

PLEASE HELP NEED ANSWER ASAPPPP!!!! WILL GIVE BRAINLIEST TO FIRST CORRECT ANSWERRRR!!!

Answers

Answer:

Remember that the division by zero is not defined, this is the criteria that we will use in this case.

1) [tex]\frac{y^2 - 1}{y} + \frac{y}{y - 3}[/tex]

So the fractions are defined such that the denominator is never zero.

For the first fraction, the denominator is zero when y = 0

and for the second fraction, the denominator is zero when y = 3

Then the fractions exist for all real values except for y = 0 or y = 3

we can write this as:

R / {0} U { 3}

(the set of all real numbers except the elements 0 and 3)

2) [tex]\frac{b + 4}{b^2 + 7}[/tex]

Let's see the values of b such that the denominator is zero:

b^2 + 7 = 0

b^2 = -7

b = √-7

This is a complex value, assuming that b can only be a real number, there is no value of b such that the denominator is zero, then the fraction is defined for every real number.

The allowed values are R, the set of all real numbers.

3) [tex]\frac{a}{a*(a - 1) - 1}[/tex]

Again, we need to find the value of a such that the denominator is zero.

a*(a - 1) - 1 = a^2 - a - 1

So we need to solve:

a^2 - a - 1 = 0

We can use the Bhaskara's formula, the two values of a are given by:

[tex]a = \frac{-(-1) \pm \sqrt{(-1)^2 + 4*1*(-1)} }{2*1} = \frac{1 \pm \sqrt{5} }{2}[/tex]

Then the two values of a that are not allowed are:

a = (1 + √5)/2

a = (1 - √5)/2

Then the allowed values of a are:

R / {(1 + √5)/2} U {(1 - √5)/2}

3/8 + 1/4 + 1/2 - 2/3 =

Answers

Answer:

[tex]\frac{11}{24}[/tex]

Step-by-step explanation:

3/8 + 1/4 + 1/2 - 2/3

- > 1/4 =  2/8

3/8 + 2/8 + 1/2 - 2/3

5/8 + 1/2 - 2/3

- > 1/2 = 4/8

5/8 + 4/8 - 2/3

9/8 - 2/3

- > LCM of 8,3: 24

- > 9/8 = 27/24

- > 2/3 = 16/24

27/24 - 16/24

11/24

Hope this helps you.

Greatest to least 2,250 2,700 2,450 2,500

Answers

Answer:

sorting;

2,7002,5002,450 2,250

greatest = 2,700

least = 2,250

HAVE A NİCE DAY

Step-by-step explanation:

GREETINGS FROM TURKEY ツ

greatest: 2,700
2,500
2,450
least: 2,250


Solve the expression using the correct order of operations.
0.75x3.2+ (9.1)2-((-2.3)-(-0.9))2

Answers

Answer:

[tex]0.75 * 3.2+ (9.1)^2-((-2.3)-(-0.9))^2 = 83.25[/tex]

Step-by-step explanation:

Given

[tex]0.75 * 3.2+ (9.1)^2-((-2.3)-(-0.9))^2[/tex]

Required

Solve

Start with the bracket

[tex]0.75 * 3.2+ (9.1)^2-((-2.3)-(-0.9))^2 = 0.75 * 3.2+ (9.1)^2-(-1.4)^2[/tex]

Evaluate all exponents

[tex]0.75 * 3.2+ (9.1)^2-((-2.3)-(-0.9))^2 = 0.75 * 3.2+ 82.81-1.96[/tex]

Evaluate all products

[tex]0.75 * 3.2+ (9.1)^2-((-2.3)-(-0.9))^2 = 2.4+ 82.81-1.96[/tex]

[tex]0.75 * 3.2+ (9.1)^2-((-2.3)-(-0.9))^2 = 83.25[/tex]

In studying the sampling distribution of the mean, you were asked to list all the different possible samples from a small population and then find the mean
of each of them. Consider the following:
Personal phone calls received in the last three days by a new employee were 2. 4, and 7. Assume that samples of size 2 are randomly selected with replacement from
this population of three values
What different samples could be chosen? What would be their sample means?
O A. Possible samples 2-4, 2-74-2: 4-7, 7-2,7-4
Sample means: 3,45,55
O B. Possible samples: 2-2.2-4,2-74-2, 4-4 4-7,7-2,7-4.7-7
Sample means: 2, 3, 4, 4.5,55,7
OC. Possible samples: 2-4 2-7, 4-7
Sample means: 3.4,45
a
Q
rd

Answers

Give more detail and I’ll help

Which triangle must be a right triangle and why?
O AA'B'C' is right because it is the image of AABC.
O AADC is right because AA' intersects AC at A.
O ABCC' is right because B lies of the line of
reflection.
O ABGC is right because G. CC')

Answers

Answer:

it would be the last one.

Step-by-step explanation:

its looking for a right triangle, a right triangle has one 90 degree angle. all of the other triagles have acute angles making them smaller than 90 degrees

Triangle BGC is the right triangle, because BG is perpendicular to CC'.

The line passing through points E, F, and G in the image is now perpendicular to the lines is DF and CG.

So we know that our triangle will be made with some of these lines.

For example, the right triangles in the figure are:

BFD, BGC, B'FD', and B'GC'.

Then, the concluded statement is ΔBGC, because BG ⊥CC.

There says that "Triangle BGC is the Right because BG is perpendicular to CC.

Learn more about right triangle here:

brainly.com/question/2217700

#SPJ7

Please explain the misleading

Answers

There are more compact cars (4*10 = 40) compared to trucks (2*10 = 20); however, the pictogram might make it appear that there are more trucks because the individual truck icon is larger compared to an individual compact car icon.

To anyone giving this image a quick glance, they may erroneously conclude that there are more trucks since their eye would notice the trucks first. Also, the person might think there are more trucks because bigger sizes tend to correspond to more proportion.

In real life, a truck is larger than a compact car, but the icons need to be the same size to have the figure not be misleading.

A very similar issue happens with the mid-size cars vs the compact cars as well. The three mid-size car icons span the same total width as the compact cars do, indicating that a reader might mistakenly conclude that there are the same number of mid-size cars compared to compact ones (when that's not true either).

Need help on this ASAP

Answers

Answer:

The answer is C

Step-by-step explanation:

The intersection of those figures results to a point

For the function, tell whether the graph opens up or opens down, identify the vertex, and tell whether the graph is wider, narrower, or the same width as the graph of y = |x|.

y = 2|x – 1| - 3



opens up, (1, 3), wider

opens up, (1, 3), narrower

opens up, (-1, -3), wider

opens up, (1, -3), narrower

Answers

Answer:

The answer is D, the last one.

9514 1404 393

Answer:

  (d)  opens up, (1, -3), narrower

Step-by-step explanation:

The factor of +2 multiplying the function tells you the graph is expanded vertically by a factor of 2. The parent function opens upward, and the positive sign on this expansion factor does not change that. The expansion means that y-values will be farther from the vertex for the same x-value distance from the vertex. This give the appearance of a narrower graph.

As always, the transformation ...

  f(x -h) +k

moves the vertex from (0, 0) to (h, k). Here, you have (h, k) = (1, -3), so that is the location of the vertex of the transformed function.

Takes a word processor 20 minutes to spell check 6 pages, how long to check 27 pages

Answers

9514 1404 393

Answer:

  90 minutes

Step-by-step explanation:

Assuming the time is proportional to the number of pages, you have ...

  time/pages = 20/6 = T/27

Multiplying by 27 gives ...

  T = 27(20/6) = 90

It is expected to take 90 minutes to check 27 pages.

CHECK MY ANSWERS PLEASE
____
The sequence is geometric:
3, 13, 23, 33,...

True
False***
_____________________
The sequence is geometric:
5, -25, 125, -625,...

True***
False

Answers

Step-by-step explanation:

For a geometric sequence,

[tex]\dfrac{a_2}{a_1}=\dfrac{a_3}{a_2}[/tex]

1. The sequence is :

3, 13, 23, 33,...

[tex]\dfrac{13}{3}\ne \dfrac{23}{13}[/tex]

It is not geometric. It is false

2. The sequence is :

5, -25, 125, -625

[tex]\dfrac{-25}{5}=\dfrac{125}{-25}\\\\-5=-5[/tex]

So, the sequence is geometric as the common ratio is same. It is true.

please help! i need this ASAP!

Answers

Answer:

C. y=7/9x+17/9

Step-by-step explanation:

Take the slope. slope= m = y2-y1/x2-x1

=5-(-2)/4-(-5)

=7/9

Then put it into point-slope form.

y-y1=m(x-x1)

=y-5=7/9(x-4)

Simplify.

y=7/9x-28/9+5

y=7/9x+17/9

Answer:

C

Step-by-step explanation:

First find the slope (change in y/ change in x) which is positive 7/9.

Then use y=mx+b and plug in the slope, and one of the given points to solve for b.

5= 7/9*4+b

5=28/9+b

5-28/9=b

45/9-28/9=b

17/9=b

Then with the slope and y intercept(b) you get the equation shown in answer c.

Hope that helps!

Use the graph to find the y-intercept and axis of symmetry

Answers

Answer: (a) and (d)

Step-by-step explanation:

From the graph, vertex is at

Graph is same about the point [tex]x=2[/tex] . Therefore, axis of symmetry is the line [tex]x=2[/tex]

Y intercept is the place where curve intersect the Y-axis that is [tex](0,2)[/tex]

Option (a) and (d) are correct.

How many cups of flour are required to make 9 dozen cookies? ( write your answer as a mixed number) Demonstrate your method of choice

Answers

Answer:

45/16

Step-by-step explanation:

[tex]1 \frac{1}{4} = 4[/tex]

[tex]x = 9 [/tex]

[tex]x = 1 \frac{1}{4} \times 9 \div 4[/tex]

[tex]x = \frac{5}{4} \times 9 \div 4[/tex]

[tex]x = \frac{45}{16} [/tex]

Find the number of integers n that satisfy n^2 < 100.​

Answers

Answer:

n=-9,-8,-7

Step-by-step explanation:

n<100

but that is the positive square root

\(-10 n is between the negative and positive square root of 100

thus, n=-9,-8,-7

The solution of the inequality n² < 100 will be less than 10.

What is inequality?

Inequality is defined as an equation that does not contain an equal sign. Inequality is a term that describes a statement's relative size and can be used to compare these two claims.

The definition of simplicity is making something simpler to achieve or grasp while also making it a little less difficult.

The inequality is given below.

n² < 100

Simplify the equation, then we have

n² < 100

n² < 10²

n < 10

The solution of the inequality n² < 100 will be less than 10.

More about the inequality link is given below.

https://brainly.com/question/19491153

#SPJ2

Other Questions
If you change the motor in your vehicle you need to notify the DMV within ____,days of this change.-20-25-10-15 The following information is given about two fixed coupon bonds from Company A and Company B, both of which have several years left until maturity. Both bonds have a par value of $1,000. Based on this information, which of the following is most accurate?Company ACompany BCoupon = 4%Coupon = 8%Yield = 6% Yield = 6%A. Company As bond is priced higher than Company Bs and Company Bs bond is traded at a premiumB. Company As bond is priced lower than Company Bs and Company Bs bond is traded at a premiumC. Company As bond is priced higher than Company Bs and Company Bs bond is traded at a discountD. Company As bond is priced lower than Company Bs and Company Bs bond is traded at a discount Describe the relationship between education and profession How many cookies will Tanya have if she bakes 12 more batches y = 70 + 18 (x) the question is on the image The following Information is avallable for the year ended December 31: Beginning raw materials inventory Raw materials purchases Ending raw materials Inventory Office supplies expense $ 4100 5,600 4,600 2,600 The amount of raw materials used in production for the year is: __________ a) $5.100 b) $8,300 c) $5,700 d) $5,600. e) $9,700 Why is the temperature constant during the melting of water? We use loops to: Process Numeric Data Ask users to enter information Repeat blocks of code Ask True/False questions when is the population of a country counted, yearly, annually, daily. Tamekia reports that the market leader in the TZX mitt category has 60 percent of all sales in the market and spends $240,000 a year, or 60 percent of all advertising expenditures, to support the brand. Under the share of market/share of voice system, Great Catch should be prepared to spend at least ________ if it hopes to achieve a market share of 30 percent. * ** Class Answer the following questions: b.write any four difference between savage and Barbarian stage of society The products obtained from hydroboration-oxidation of cis-2-butene are identical to the products obtained from hydroboration-oxidation of trans-2-butene. Draw the products and explain why the configuration of the starting alkene is not relevant in this case. Anderson has $120 and is spending $8 a day. Kate has $60 and is saving $7 a day. After how many days will they have the same amount of money? a person who has a vegatarin plan Please help! Im confused. 2. Why can't a ball roll continuously on a rough surface A. The rough surface is not made for balls to roll. B. The force which enables the ball to move is released. C. The rough texture of the surface blocks the movement of the D. The rough texture of the surface permits the movement of th 3. What eventually happens to a moving object when it movese A. It will continuously move. B. It will stop immediately. C. It will eventually slow down and stop. D. It will move either to the right or left side and stop. Sanders Company has a debit balance of $7,000 in its Allowance for Doubtful Accounts before any adjustments are made. Based on a review of its accounts receivable at the end of the year, Sanders estimates that $70,000 of its receivables are uncollectible. The amount of bad debt expense which should be reported for the year is $70,000. $7,000. $77,000. $63,000. $70,000. If three children equally share half loaf of bread among themselves, what fraction will each get? f the essay question was, What is Stephen Kings best work? which of the following would be the best way to start a conclusion paragraph? if you were pouring juice into a cylindrical cup with a 2 inch radius and height of 8 inches. you want to fill it exactly half full, how many cubic inches of juice would you pour